CVS Healio Cases Flashcards
(148 cards)
An 87-year-old woman is admitted with pneumonia,
complicated by septic shock. She develops atrial fibrillation with an
uncontrolled ventricular response. Her temperature is 101.8 F, blood pressure
80/40 mm Hg, respirations 20 per minute and heart rate 148 beats per minute.
Prior to the atrial fibrillation, her blood pressure was similarly low. Which
of the following is the next appropriate step in management?
A. Intravenous metoprolol
B. Intravenous diltiazem
C. Intravenous amiodarone
D. Emergent direct current cardioversion
E. Emergent chemical cardioversion with ibutilide
Correct Answer: C. Intravenous amiodarone
Explanation: When a patient is hypotensive due to atrial fibrillation
then emergent cardioversion is necessary; however
A 24-year-old female presents with dyspnea and edema. She
is noted to have a holosystolic murmur at the left lower sternal border as well
as a short early diastolic murmur at the same location. An echocardiogram
reveals the presence of a supracristal ventricular septal defect. Which of the
following explains the diastolic murmur?
A. Aortic regurgitation
B. Mitral stenosis
C. Cor triatriatum
D. Ebstein’s anomaly
E. Pulmonic regurgitation
Correct Answer: A. Aortic regurgitation
Explanation: Supracristal ventricular septal defects occur just beneath the aortic valve at the left ventricular outflow tract. A Venturi effect can occur from the left to right shunt
causing the aortic valve leaflet to prolapse into the ventricular septal defect and resulting in significant aortic valve regurgitation.
A 57-year-old man with no cardiovascular past medical
history presents with acute onset palpitations and intermittent dizziness. He
presents to the emergency department 8 hours after the onset of symptoms and
has never had symptoms like this before. He is found to have atrial
fibrillation with an uncontrolled ventricular response (heart rate 170 beats
per minute). An ECG is completely normal, as is laboratory evaluation with a
basic metabolic panel, complete blood count and a thyroid stimulating hormone
level. Which of the following is a reasonable strategy?
A. Direct current cardioversion with aspirin alone for
thromboembolism prophylaxis
B. Direct current cardioversion with full anticoagulation
for 3 to 4 weeks afterwards for thromboembolism prophylaxis
C. Transesophageal echocardiography to exclude a left atrial
appendage thrombus followed by direct current cardioversion and aspirin alone
for thromboembolism prophylaxis
D. Transesophageal echocardiography to exclude a left atrial
appendage thrombus followed by direct current cardioversion and full
anticoagulation for 3 to 4 weeks afterwards for thromboembolism prophylaxis
Correct Answer: A. Direct current cardioversion with aspirin alone for
thromboembolism prophylaxis
Explanation: When the duration of atrial fibrillation is thought to be less than 48 hours
transesophageal echocardiography is not required to evaluate for left atrial appendage thrombus. Direct current cardioversion can safely be performed with very low risk for thromboembolism in this situation.
A 58-year-old woman with asthma and a history of
hypertension, type 2 diabetes and a prior stroke presents to the emergency
department with substernal chest pains radiating to her left arm. She is short
of breath, diaphoretic and nauseated. She is afebrile with a blood pressure of
150/90 mm Hg, heart rate of 90 beats per minute, respirations 22 per minute and
oxygen 92% on room air. Physical examination reveals an S4 heart sound and
significant wheezing on lung examination. Her ECG is below:
Enlarge
Image: Learn the Heart
Which of the following combinations of initial medical
therapy is appropriate?
A. Aspirin, clopidogrel, unfractionated heparin,
beta-blocker, oxygen, nitroglycerin
B. Aspirin, prasugrel, low molecular weight heparin,
non-dihydropyridine calcium channel blocker, oxygen, nitroglycerin
C. Aspirin, clopidogrel, low molecular weight heparin,
non-dihydropyridine calcium channel blocker, oxygen, nitroglycerin
D. Aspirin, prasugrel, unfractionated heparin, beta-blocker,
oxygen, nitroglycerin
Correct Answer: C. Aspirin
non-dihydropyridine calcium channel blocker
oxygen
Explanation: The non-dihydropyridine calcium channel blockers diltiazem and verapamil can be used when there is a contraindication to beta-blockers (such as asthma) and there is no heart failure or significant left ventricular systolic dysfunction present. They are especially helpful to lower heart rate and reduce oxygen demand in this situation. Sublingual nifedipine is contraindicated due to a reflexive increase in the sympathetic nervous system
which can be harmful.
Recall that prasugrel is contraindicated if a prior stroke or transient ischemic attack is present. Also
either low molecular weight heparin or unfractionated heparin are considered reasonable choices for anticoagulation.
A 58-year-old man is undergoing a treadmill ECG stress
test for evaluation of chest pain. After 6 minutes on a Bruce protocol, he
develops substernal chest pain radiating to his left arm and there is 4 mm of
ST depression in leads V5 and V6. His vital signs are stable. Which of the
following would you expect to find on his physical examination?
A. An S3 heart sound and a systolic ejection murmur
B. An S3 heart sound and a holosystolic murmur at the
cardiac apex
C. An S4 heart sound and a systolic ejection murmur
D. An S4 heart sound and a holosystolic murmur at the
cardiac apex
Correct Answer: D. An S4 heart sound and a holosystolic murmur at the
cardiac apex
Explanation: Physical examination findings are relatively non-specific usually only present during the anginal episode making this a less helpful means of diagnosis. When examined during an anginal attack
the heart rate and blood pressure may be elevated due to increased sympathetic tone. An S4 heart sound may be present during myocardial ischemia due to the lack of adenosine triphosphate production impairing left ventricular relaxation. Recall that myocardial relaxation is an active process requiring adenosine triphosphatewhich is reduced during ischemia and a S4 heart sound occurs when a non-compliant
During inferior ischemia
posteromedial papillary muscle dysfunction can cause mitral regurgitation resulting in a holosystolic murmur at the cardiac apex radiating to the axilla. This rarely occurs during anterior or lateral ischemia since the anterolateral papillary muscle has dual supply from the left anterior descending and circumflex coronary artery.
When the left ventricular end-diastolic pressure increases during myocardial ischemia
that pressure can be transmitted backward to the pulmonary veins and into the pulmonary vasculature causing transient pulmonary edema resulting in dyspnea and rales on lung examination.
A 38-year-old woman presents with the acute onset of
palpitations. Her 12-lead ECG is below. She is afebrile with a heart rate of
180 beats per minute, blood pressure of 120/80 mm Hg and respirations of 18 per
minute. Which of the following is an appropriate course of action?
Enlarge
A. Emergent direct current cardioversion
B. Adenosine 6 mg intravenously, followed by a saline flush
C. Intravenous procainamide
D. Intravenous amiodarone
Correct Answer: B. Adenosine 6 mg intravenously
Explanation: When a narrow QRS complex tachycardia is present
and the diagnosis is unclear
There is no need for emergent direct current cardioversion
as the patient is hemodynamically stable. Antiarrhythmic drug therapy is not recommended until the rhythm is diagnosed. Procainamide is not commonly used
A 58-year-old man presents to the emergency department
after experiencing intermittent chest pain for 2 days. He describes substernal
chest pressure radiating to his jaw. He is short of breath and diaphoretic. His
temperature is 100 F, blood pressure is 140/90 mm Hg, respirations are 22
breaths per minute, heart rate is 70 beats per minute and oxygen is 92% on room
air.
He is taken for emergency percutaneous coronary
intervention. This discovers thrombus in the mid-left anterior descending
coronary artery, and primary percutaneous coronary intervention is performed;
however, there was sluggish forward flow afterwards despite the left anterior
descending artery remaining widely patent.
After a diagnosis ofanterior ST-elevation
myocardial infarction complicated by no-reflow, he is followed and discharged
home. During a clinic follow-up visit 6 weeks later, he is diagnosed with left
ventricular aneurysm.
Four months later, he calls 911 due to sudden onset of
weakness and pre-syncope. He is short of breath, diaphoretic and feels
significant palpitations. He is afebrile with a blood pressure of 80/40 mm Hg,
heart rate of 240 beats per minute, respirations 26 breaths per minute and
oxygen 88% on room air. His ECG is below:
Enlarge
Image: Learn the Heart
Which of the following is the correct diagnosis?
A. Ventricular fibrillation
B. Monomorphic ventricular tachycardia
C. Polymorphic ventricular tachycardia
D. Wolff-Parkinson-White with pre-excitation and atrial
fibrillation
Correct Answer: B. Monomorphic ventricular tachycardia
Explanation: See theVentricular Tachycardia ECG Reviewfor a complete summary.
The four main complications of a left ventricular aneurysm include the following:
Ventricular tachycardia: The scar within the left ventricular aneurysm is a focus for ventricular arrhythmias which can lead to sudden cardiac death.
Heart failure: The portion of the heart that contains the aneurysm is not contractile and is frequently “dyskinetic.” This results in overall decrease in heart function and the development of congestive heart failure.